LSAT and Law School Admissions Forum

Get expert LSAT preparation and law school admissions advice from PowerScore Test Preparation.

 Administrator
PowerScore Staff
  • PowerScore Staff
  • Posts: 8917
  • Joined: Feb 02, 2011
|
#38951
Complete Question Explanation
(The complete setup for this game can be found here: lsat/viewtopic.php?t=15134)

The correct answer choice is (D)

If P performs before S, Template 1 already provides the relevant solution.

Answer choice (A) is not necessarily true, because V could also perform first in Template 1.

Answer choice (B) is not necessarily true, because P could perform first in Template 1.

Answer choice (C) is not necessarily true, because S could be the second to perform after P.

Answer choice (D) is the correct answer choice, because S must performs after either P or T, but not after both. Thus, if P performs before S, we need to ensure that T does not also perform before S. Therefore, T must perform after S. The resulting P S T sequence proves answer choice (D) to be correct.

Answer choice (E) is not necessarily true, because the order of S and K in Template 1 is unknown.

Get the most out of your LSAT Prep Plus subscription.

Analyze and track your performance with our Testing and Analytics Package.